Nichtstationäre Lösungen für stationäre ARMA-Gleichungen


8

Mit "stationär" meine ich "schwach stationär".

Betrachten Sie eine "stationäre" AR (1) -Gleichung:

Xt=φXt1+εt,
wobei diskrete Zeitmomente sind, ein null-mittleres weißes Rauschen (nur eine iid-Sequenz), . Es ist bekannt, dass es eine stationäre Lösung gibt (dh eine diskrete Zeitreihe, die die Gleichung erfüllt). Bezeichne es mitWir können jedoch eine andere Zeitreihe einführen , die eine instationäre Lösung für die "stationäre" Gleichung zu sein scheint (klar ist nicht frei von , da offensichtlich ist Null Bedeutung).tZεtφ(1,1)Xt.Yt=Xt+φtE[Yt]tXt

Ist es bei einem allgemeineren stationären AR ( ) -Prozess möglich, die schwache Stationaritätseigenschaft irgendwie zu beschädigen? Oder ist es im Allgemeinen wahr, dass jede stationäre zeitdiskrete AR- (oder sogar ARMA-) Gleichung eine nichtstationäre Lösung hat?p


Könnte ein bisschen erweitern? Können Sie erklären, wie eine instationäre Lösung für zu sein scheint ? (Vielleicht wäre es eine bessere Praxis, und in derselben Übung zu verwenden, da beide "phi" sind, was es verwirrend machen kann.)Yt=Xt+ϕtXt=φXt1+εtϕφ
Richard Hardy

Was meinst du mit einer "Lösung", was ist das für ein Objekt? (Wie eine Konstante, ein stochastischer Prozess, ...) Könnten Sie das näher erläutern, vielleicht diesen Abschnitt des Beitrags erweitern?
Richard Hardy

Richard, die Lösung wird natürlich als Zeitreihe angenommen. Ich habe es dem Beitrag hinzugefügt.
Nikita

Können Sie ganz oder teilweise zeigen, dass eine (nichtstationäre) Lösung ist? Vielleicht bin ich auch wählerisch, aber ich bin nicht so an die Terminologie und daher ist es etwas verwirrend , in der allgemeinen Form AR (1) -Gleichung UND als Lösung dafür zu haben. Könnten wir die beiden irgendwie notational unterscheiden? (Aber vielleicht ist es Standard, eine solche Notation zu verwenden, dann ignorieren Sie einfach meinen Kommentar.)YtXt
Richard Hardy

Ich meinte, dass die Tatsache, dass eine Lösung ist, leicht zu überprüfen ist. Yt
Nikita

Antworten:


0

Wenn Sie Ihren Prozess fortsetzen, werden Sie feststellen, wie der Begriff verschwindet: φt

limtφt=0

Also, obwohl die rechte Seite von endlichem abhängig ist .

E[Yt]=E[Xt]+E[φt]=t0
t

So ist die Antwort auf Ihre Frage ist , dass Ihr Prozess ist nicht nicht-stationär. Daher dient es nicht als Gegenbeispiel.Yt

Zusätzliche Gedanken . Sie haben Ihre Frage nach Lösungen der stochastischen Prozesse formuliert . Schauen Sie sich an, was die Lösung des AR (1) -Prozesses ist.

Wenn Sie beispielsweise Schritte voraussagen, erhalten Sie:τ

Xt+τ=φτ(Xt+s=1τεt+sφs)

Sie können sehen, wie es einfach zu dem Rauschen um Null zusammenfällt, wenn wächst, unabhängig davon, was das anfängliche . Wenn Sie Ihren Term hinzufügen, verschwindet er ebenfalls, sodass die stabile Lösung dieselbe ist: Rauschen um Null:τXtφτ

Xt+τ+φτ=φτ(Xt+s=1τεt+sφs+φs)

Beantwortet das die Fragen? Ich habe Probleme, die Verbindung klar zu erkennen (obwohl ich eine Verbindung sehe ).
Richard Hardy

Ich habe die Antwort aktualisiert.
Aksakal

Vielen Dank. Ich finde sowohl die Frage als auch die Antwort interessant, aber es erfordert einige Mühe, meinen Kopf um sie zu wickeln. Element für Element sind diese einfach, aber die Beziehungen zwischen ihnen können täuschen :)
Richard Hardy

@ RichardHardy, ich bin hier faul. Vielleicht hätte ich das alles im Rahmen von SDE beschreiben sollen, dann wäre es klarer.
Aksakal

Dürfen wir auch den Fall und eine Art von Handwellen-endlichem (insbesondere in der Erwartung in der zweiten Formel)? Ich kann sehen, wohin Sie gehen, wenn Sie sich die Fixpunkte von SDEs ansehen, aber ist das wirklich das, was wir hier brauchen? Ich denke, das hängt von der Definition einer Lösung ab , und das OP scheint nicht an Fixpunkten interessiert zu sein, sondern an Prozessen, die eine Eigenschaft / Gleichung erfüllen (siehe die Kommentare unter dem OP). tt
Richard Hardy

0

Die in der Frage verwendete Terminologie ist nicht ganz richtig. Sie verwechseln das Modell (oder die Gleichungen) und die Lösung des Modells.

Es ist nicht sinnvoll, von einer stationären oder instationären Gleichung (in diesem Fall einem System stochastischer Differenzgleichungen) zu sprechen. Die fehlende Stationarität ist eine Eigenschaft einer Lösung. Eine Gleichung kann stationäre oder instationäre Lösungen haben.

Was Sie gefunden haben, sind zwei Lösungen, eine stationäre und eine instationäre, für die AR (1) -Gleichung, wenn der AR-Parameter . (Wenn , Ersatz für in Ihrem Beispiel) . Im Gegensatz dazu, wenn gibt es nur instationäre Lösungen.|ϕ|1|ϕ|>1tt|ϕ|=1

Die Antwort auf Ihre Frage lautet: Ja, dies verallgemeinert sich auf den Fall AR (p). Die AR (p) -Gleichung (en) hat sowohl stationäre als auch instationäre Lösungen, wenn das Polynom keine Wurzeln auf dem Einheitskreis hat und alle Wurzeln real sind.

Φ(L)Xt=ϵt,t=1,0,1,
Φ(zp)

Angenommen, das AR (2) -Modell hat eine stationäre Lösung und hat zwei reelle Wurzeln und , dann ist eine instationäre Lösung.

Xt=ϕ1Xt1+ϕ2Xt2+ϵt
(Xt)z2ϕ1zϕ2ab
Xt+at+bt1

Wenn Sie einstellen und berücksichtigen, wird Ihr AR (1) wiederhergestellt.ϕ2=0zϕ1=0

Durch die Nutzung unserer Website bestätigen Sie, dass Sie unsere Cookie-Richtlinie und Datenschutzrichtlinie gelesen und verstanden haben.
Licensed under cc by-sa 3.0 with attribution required.